Talk:2005 AIME II Problems/Problem 11

Revision as of 11:15, 21 July 2006 by JBL (talk | contribs)
(diff) ← Older revision | Latest revision (diff) | Newer revision → (diff)

This question can't possibly be right. If $a_{k + 1} = a_{k - 1} - \frac 3{a_k}$, $a_0= 37$ and $a_1 = 72$, it most certainly can't be a sequence of integers. I also can't find an alternative problem statement anywhere. --JBL 11:15, 21 July 2006 (EDT)